LSAT and Law School Admissions Forum

Get expert LSAT preparation and law school admissions advice from PowerScore Test Preparation.

 Blueballoon5%
  • Posts: 156
  • Joined: Jul 13, 2015
|
#44508
Hello! I've read through the different explanations for why answer choice B is wrong, but I am still a bit confused. I hope you can help clarify.

The reason that I am confused is because, according to the explanations above, answer choice B leaves open the possibility of another planet in the solar system besides Alpha. A hypothetical planet Beta may exist and I understand that this could leave open the possibility of the moons in the solar system all orbiting planet Beta. However, doesn't the conclusion of the stimulus already establish that planet Alpha has all the moons? It is impossible for planet Beta to have even one moon because the conclusion already states that planet Alpha has them all. Rather than suggesting an existence of another planet, answer choice B instead seems to reinforce the idea that the moons all orbit the same planet (Alpha).

Thus, when we combine the "Premise (Any moon, by definition, orbits some planet in a solar system) + Answer Choice B (Every moon in S4 orbits the same planet) = Conclusion (So, the moons in solar system S4 all oribit the planet Alpha)"

I hope you can help me! (p.s. I did get the right answer because I thought answer choice C was much stronger, but I am still a bit confused how I can rule out answer choice B :) )
 Shannon Parker
PowerScore Staff
  • PowerScore Staff
  • Posts: 147
  • Joined: Jun 08, 2016
|
#44535
Blueballoon5% wrote:Hello! I've read through the different explanations for why answer choice B is wrong, but I am still a bit confused. I hope you can help clarify.

The reason that I am confused is because, according to the explanations above, answer choice B leaves open the possibility of another planet in the solar system besides Alpha. A hypothetical planet Beta may exist and I understand that this could leave open the possibility of the moons in the solar system all orbiting planet Beta. However, doesn't the conclusion of the stimulus already establish that planet Alpha has all the moons? It is impossible for planet Beta to have even one moon because the conclusion already states that planet Alpha has them all. Rather than suggesting an existence of another planet, answer choice B instead seems to reinforce the idea that the moons all orbit the same planet (Alpha).

Thus, when we combine the "Premise (Any moon, by definition, orbits some planet in a solar system) + Answer Choice B (Every moon in S4 orbits the same planet) = Conclusion (So, the moons in solar system S4 all oribit the planet Alpha)"

I hope you can help me! (p.s. I did get the right answer because I thought answer choice C was much stronger, but I am still a bit confused how I can rule out answer choice B :) )
You answered your own question here but you did not realize it. In a justify question you are looking for a answer choice that is sufficient to bring about the conclusion. You cannot rest on the information given in the conclusion for the answer. You must look at the answer choice that combined with the premise or premises in the stimulus bring about the conclusion.

For example, if i asked you to justify the following: Premise: Every pitcher can throw 90 miles per hour. Conclusion: Justin can throw 90 miles per hour. Justification: Justin is a pitcher. You add "Justin is a pitcher" to "every pitcher can throw 90 miles per hour" and you know that "Justin can throw 90 miles per hour."

Here we have the premise: Any moon, by definition, orbits some planet in a solar system.
If we add answer choice B: "Every moon in S4 orbits the same planet," it does not lead to the conclusion that every moon in S4 orbit Alpha because there could be another planet.

This is why C seemed stronger to you. Because it is the only answer choice that is sufficient to lead to the conclusion.

I hope this makes sense.
Shannon
 Blueballoon5%
  • Posts: 156
  • Joined: Jul 13, 2015
|
#44542
Thanks Shannon! I understand now! :-D
 esther913
  • Posts: 23
  • Joined: Apr 13, 2019
|
#64246
Hi,
I know that this is a Justify The Conclusion type question, but the task seemed more like finding a supporter assumption for the conclusion.
I diagrammed the stimulus as :

moon :arrow: orbit some planet in solar system

conclusion: moons in S4 :arrow: orbit around alpha

I thought there was a missing link between the two necessary conditions, so I chose (c) -Alpha is the only planet in S4- to close the gap. However, I am confused because I thought linking rogue elements and closing logical gaps are what supporter assumptions do. If the correct answer choice for a Justify type question acts like a supporter assumption, how can I tell which is which? :cry:

One more question: Can I say that (A) is wrong because it mentions only one moon, while the conclusion is about several moons?

Thank you.
 Jay Donnell
PowerScore Staff
  • PowerScore Staff
  • Posts: 144
  • Joined: Jan 09, 2019
|
#64250
Hi Esther, great question!

In a sense, a Justify answer works very much like a Supporter, but in an extremely powerful way. The correct answer to a Justify question must "bridge the gap" between the relevant premises and the conclusion, so I personally always refer to Justify answers not as Supporters, but as Provers.

A Justify is essentially a Super Strengthen question. Where the correct answer in a Strengthen question offers a fact that provides a great deal of support to the conclusion in the stimulus, the answer to a Justify question brings in sufficient evidence (aka a really strong fact!) to completely prove the conclusion in the stimulus. This is why a Justify question technically provides in its correct response a sufficient assumption, or an assumption that is enough to prove/justify the conclusion with its insertion into the argument.

That's why the best way to check your answer in a Justify question is to apply the Justify Formula, which is to literally insert the answer into the argument as an additional premise, and see if that answer with the original premises together prove that the conclusion must follow.

Here, we also get to use the Mechanistic Approach in that there is a new concept introduced into the conclusion which is the source of the current problem in the argument, so in order to ensure the argument becomes valid we must "support the unsupported," or basically make sure the correct response contains or covers the new term.

That 'new term' in this argument is actually two items: the planet Alpha and S4. Any answer that doesn't contain both terms is immediately eliminated because if we don't bring in support for the planet Alpha and S4 in the correct response, then they remain unsupported and the argument remains flawed.

That right there kills answer choice A. Knowing there is only one moon does nothing to 'bridge the gap' here, because we have no idea how many planets are in this S4 solar system. Maybe that one moon orbits Planet Beta, or even Planet Esther?

C works so well here because if we know that Alpha is the only planet in S4, and the premise told us that moons must orbit some planet, then we have sufficiently proven (justified) the conclusion that any moon in S4 must orbit the planet Alpha.


In short, Justify answers can actually function like Supporters, but they are much more than that. Think of them like Provers, and use that Justify Formula to double check that the answer brings in enough force to 100% prove/justify that conclusion.

Hope that helps!

Happy Friday :-D 8-)
 esther913
  • Posts: 23
  • Joined: Apr 13, 2019
|
#64291
Dear Jay,
Your explanation was immensely helpful!
Thank you so much for taking the time to explain things thoroughly.
Now I understand how the correct answer works. Thank you :-D

Get the most out of your LSAT Prep Plus subscription.

Analyze and track your performance with our Testing and Analytics Package.